Difference between revisions of "2016 AIME I Problems/Problem 13"
Fclvbfm934 (talk | contribs) (Created page with "Notice that we don't really care about what the <math>x</math>-coordinate of the frog is. So let's let <math>f(y)</math> denote the expected number of times Freddy will jump a...") |
Fclvbfm934 (talk | contribs) |
||
Line 1: | Line 1: | ||
+ | |||
+ | ==Solution== | ||
Notice that we don't really care about what the <math>x</math>-coordinate of the frog is. So let's let <math>f(y)</math> denote the expected number of times Freddy will jump at a <math>y</math> coordinate of <math>y</math> until he reaches the line <math>y = 24</math>. So therefore we want to find <math>f(21)</math>. | Notice that we don't really care about what the <math>x</math>-coordinate of the frog is. So let's let <math>f(y)</math> denote the expected number of times Freddy will jump at a <math>y</math> coordinate of <math>y</math> until he reaches the line <math>y = 24</math>. So therefore we want to find <math>f(21)</math>. | ||
Line 8: | Line 10: | ||
<cmath>f(k) = z - k - 2k^2 </cmath> | <cmath>f(k) = z - k - 2k^2 </cmath> | ||
Now, we also have that <math>f(24) = 0</math> so that gives us <math>f(24) = z - 24 - 2 \cdot 576 = 0</math> so <math>z = 1176</math>. So now we know <math>f(k) = 1176 - k - 2k^2</math> so plugging in <math>k = 21</math> we get <math>f(21) = 1176 - 21 - 882 = \boxed{273}</math> | Now, we also have that <math>f(24) = 0</math> so that gives us <math>f(24) = z - 24 - 2 \cdot 576 = 0</math> so <math>z = 1176</math>. So now we know <math>f(k) = 1176 - k - 2k^2</math> so plugging in <math>k = 21</math> we get <math>f(21) = 1176 - 21 - 882 = \boxed{273}</math> | ||
+ | |||
+ | == See also == | ||
+ | {{AIME box|year=2015|n=I|before=First Problem|num-a=2}} | ||
+ | {{MAA Notice}} |
Revision as of 16:00, 4 March 2016
Solution
Notice that we don't really care about what the -coordinate of the frog is. So let's let denote the expected number of times Freddy will jump at a coordinate of until he reaches the line . So therefore we want to find .
So we have . Suppose Freddy is at . He has a probability of moving horizontally, chance of moving up and chance of moving down. So we have So we get the recursion . Rearranging we see . That means the difference between consecutive terms goes down by each time. So for convenience let's let and . So that means Yes, this is a quadratic. Now, notice that since there is a boundary, we have to give special care to . We have so this turns into and this results in . So now we know Now, we also have that so that gives us so . So now we know so plugging in we get
See also
2015 AIME I (Problems • Answer Key • Resources) | ||
Preceded by First Problem |
Followed by Problem 2 | |
1 • 2 • 3 • 4 • 5 • 6 • 7 • 8 • 9 • 10 • 11 • 12 • 13 • 14 • 15 | ||
All AIME Problems and Solutions |
The problems on this page are copyrighted by the Mathematical Association of America's American Mathematics Competitions.